2500 feet is to how many meters

Answers

Answer 1

Answer:  is 762 meters

Step-by-step explanation:

One foot is equal to 0.3048 meters so multiply 2500 by 0.3048 to find how many meters there are.

2500 * 0.3048 = 762

Answer 2

Answer:762.5 meters

Step-by-step explanation:

1 foot =0.305 m

Hence,0.305×2500=762.5 metres as the answer.

2500 Feet Is To How Many Meters

Related Questions

Find the value of x.

Answers

Answer:

x = 10

Step-by-step explanation:

(segment piece) x (segment piece) =         (segment piece) x (segment piece)

6*x = 12*5

6x = 60

Divide by 6

6x/6 = 60/6

x = 10

Answer:

[tex]\huge\boxed{x = 10}[/tex]

Step-by-step explanation:

According to chord-chord theorem:

(x)(6) = (5)(12)

6x = 60

Dividing both sides by 6

x = 10

[tex]-6\left(1-m\right)=9-2m[/tex]

Answers

Answer:

[tex]m=\frac{15}{8}[/tex]

Step-by-step explanation:

[tex]-6\left(1-m\right)=9-2m[/tex]

[tex]-6(1-m)=-6+6m[/tex]

[tex]-6+6m=9-2m[/tex]

Add 6 to both sides:-

[tex]-6+6m+6=9-2m+6[/tex]

[tex]6m=-2m+15[/tex]

Now, add 2m to both sides:-

[tex]6m+2m=-2m+15+2m[/tex]

Divide both sides by 8:-

[tex]\frac{8m}{8}=\frac{15}{8}[/tex]

[tex]m=\frac{15}{8}[/tex]

-----------------------

OAmalOHopeO

-----------------------

Answer:

your answer is m=15/8

Briefly explain how you would graph an equation such as y=7x-2

Answers

Answer:

put a mark at negative 2 on the graph then you would go up seven rigght one until you cant then you would go back to -2 and go down seven left one until you cant

Step-by-step explanation:

You pack sandwiches for a hike with your friends. Each sandwich takes 2 slices of bread, and each hiker eats one sandwich.

How many slices of bread are used for n hikers?

Answers

Given that:-

→ 1 sandwich = 2 slices of bread.

→ 1 hiker = 1 sandwich.

Then we have to find number of bread slices for n hikers .

→ Number of bread slices for 1 hiker = 2

→ Number of bread slices for 2 hikers = 2 × 2

→ For 3 hikers = 3 × 2

So in similar way

Number of bread slices for n hikers = 2×n 2n

So 2n is the answer.

1. Look at the numbers listed below. Which numbers are rational? Select all that apply.

9.1874639...

424, 489

121

5/3​

Answers

I believe all of them are rational.


“The rational numbers include all the integers, plus all fractions, or terminating decimals and repeating decimals. Every rational number can be written as a fraction a/b, where a and b are integers”

What is the domain of the relation (8, -2), (4,-2), (3, 2), (-5, -3)?
A. {8,4,3, -5}
B. {-8, -4, 3, 5)
C. 2-5, -3, 4, 8}
D. 2-3, -2, 2}

Answers

Answer:

A. {8, 4, 3, -5}

Step-by-step explanation:

The domain is the list of x values in a given function. Therefore, the domain is {8, 4, 3, -5}.

Write a slope-intercept equation for a line passing through the point (5,-5) that is parallel to the line x = -2. Then write a second equation for a line passing through the point (5,-5) that is perpendicular to the line x=-2.

Answers

Answer:

1. y=-2x+5

2. y=1/2x-7.5

Step-by-step explanation:

you plug in the cordinates for the y intercept and you already have the slope.

y=mx+b

m= slope which is -2

which value of -7(x2-6)+2y when x=2 and y=6

Answers

Answer:

Solution

= -7 (x2 -6) +2y

= -7 ( 2*2 - 6 ) + 2 * 6

= -7 ( 4 - 6) + 12

= -7 - 2 + 12

= - 9 + 12

= 3

I hope this help u :)

Using the right triangle below, find the tangent of angle B.

A
600
0
3-13
300
B

Answers

Answer:1&3

Step-by-step explanation:

plz help me f(x) = 2x – 2 , find f(– 1)

Answers

Answer:

-4

Step-by-step explanation:

f(-1) = 2(-1) - 2 = -4

Plug in the f(-1) into f(x)=2x-2, which is f(-1)=2(-1)-2 and the answer is-4

Find the value of x if 2x-1÷3 is greater than 1-3x÷4 by 1​

Answers

[tex]\\ \sf\longmapsto \dfrac{2x-1}{3}-\dfrac{1-3x}{4}=1[/tex]

[tex]\\ \sf\longmapsto \dfrac{4(2x-1)-3(1-3x)}{12}=1[/tex]

[tex]\\ \sf\longmapsto 8x-4-3+9x=12(1)[/tex]

[tex]\\ \sf\longmapsto 17x-7=12[/tex]

[tex]\\ \sf\longmapsto 17x=12+7[/tex]

[tex]\\ \sf\longmapsto 17x=19[/tex]

[tex]\\ \sf\longmapsto x=\dfrac{19}{17}[/tex]

2x-1/3+1=1-3x/4
5x-1/3=4
5x-1=12
5x=13
X=2.6

Simplify 18 - 2[x + (x - 5)].
A) 13 - x
B) 13 - 4x
C) 8 - 4x
D) 28 - 4x

Answers

Answer:

Well the correct answer is −4x+28... But I don't

see that as one of your options. :/

Simplify:

18−2(x+x−5)

Distribute:

=18+(−2)(x)+(−2)(x)+(−2)(−5)

=18+−2x+−2x+10

Combine Like Terms:

=18+−2x+−2x+10

=(−2x+−2x)+(18+10)

=−4x+28

Answer:

−4x+28

Answer:

[tex] \boxed{ \bold{ \huge{ \boxed{ \sf{28 - 4x}}}}}[/tex]

Option D is the correct option.

Step-by-step explanation:

18 - 2 [ x + ( x - 5 ) ]

Remove the unnecessary Parentheses

⇒18 - 2 [ x + x - 5 ]

Collect like terms

⇒18 - 2 [ 2x - 5 ]

Distribute 2 through the parentheses

⇒18 - 4x + 10

Add the numbers

28 - 4x

Hope I helped!

Best regards!!

in angle PQR, angleQ=70 angle R=45 then angleP=.....

Answers

Answer:

65

Step-by-step explanation:

Because its a triangle,

P + Q + R = 180

So,

P + 70 + 45 = 180

P + 115 = 180

Subtract 115 on both sides

P = 65

Multiply negative 1 over 3 multiplied by 1 over 4. Which of the following is correct? negative 1 over 3 negative 1 over 4 negative 1 over 7 negative 1 over 12

Answers

Answer:

-1/12

Step-by-step explanation:

-1/3 * 1/4

Multiply the numerators

-1 *1 = -1

Multiply the denominators

3 *4 =12

numerator over denominator

-1/12

Answer:

the answer is [tex]\frac{-1}{12} \\[/tex]

Step-by-step explanation:

[tex]\frac{1}{3} x \frac{1}{4}[/tex]

3x4= 12 is now the denominator

                       and

the numerator stays the same because -1x1=-1

so there for the answer is [tex]\frac{-1}{12}[/tex]

hope this helped you  ^.^ happy holiday

5000 x 10 x 10 x 50​

Answers

Answer:

5000 x 10 x 10 x 50​ = 25000000

Step-by-step explanation:

you can just use a calculator....

Answer:

25000000

Step-by-step explanation:

Use linear regression to predict the length of a 40-month-old animal. The ages and lengths of several animals of the same species are recorded in the following table: (2 points)

Answers

Answer:

Aw what animal

Step-by-step explanation:

Srry I need pointse

please help i'm having a very hard time understanding this!

Answers

Hi there!

check the pic

#YoghurtCup!

Answer: F

Step-by-step explanation:

Put all numbers with 1 exponent.

[tex](\sqrt{8^4} *\sqrt{6^4} )^{\frac{3}{2} }\\\\=(2^{\frac{3*4}{2} }*(2^{\frac{4}{2} }*3^{\frac{4}{2}} )\ )^{\dfrac{3}{2} }\\\\\\=(2^6*2^2*3^2)^{\dfrac{3}{2} }\\\\=2^{8*\frac{3}{2} }*3^{2*\frac{3}{2}} \\\\\\\boxed{=2^{12}*3^3}\\\\\\Answer\ F[/tex]

if n(p(a)) = 256. find n(a)​

Answers

n(p(a)) = 256

256 = 2^8

n(a) = 8

Must click thanks and mark brainliest

the set is points indicated by 12.is the ___ of the angle

Answers

Answer:

Vertex

Step-by-step explanation:

The vertex is the point were two rays meet to form an angle.

If my answer is incorrect, pls correct me!

If you like my answer and explanation, mark me as brainliest!

-Chetan K

congruent complements theorem

Answers

Answer:

Congruent Complements Theorem If 2 angles are complementary to the same angle, then they are congruent to each other.

HOPE THIS HELPS

MARK IT BRAINLIEST!!!!

Given the following three points, find by hand the quadratic function they represent.
(-1,-8), (0, -1),(1,2)
(1 point)
Of(x) = -51% + 87 - 1
O f(x) = -3.2? + 4.1 - 1
Of(t) = -202 + 5x - 1
Of(1) = -3.1? + 10.1 - 1​

Answers

Answer:

The correct option is;

f(x) = -2·x² + 5·x - 1

Step-by-step explanation:

Given the points

(-1, -8), (0, -1), (1, 2), we have;

The general quadratic function;

f(x) = a·x² + b·x + c

From the given points, when x = -1, y = -8, which gives

-8 = a·(-1)² + b·(-1) + c = a - b + c

-8 =  a - b + c.....................................(1)

When x = 0, y = -1, which gives;

-1 = a·0² + b·0 + c = c

c = -1.....................................................(2)

When x = 1, y = 2, which gives;

2 = a·1² + b·1 + c = a + b + c...............(3)

Adding equation (1) to (3), gives;

-8 + 2 = a - b + c + a + b + c

-6 = 2·a + 2·c

From equation (2), c = -1, therefore;

-6 = 2·a + 2×(-1)

-2·a  = 2×(-1)+6 = -2 + 6 = 4

-2·a = 4

a = 4/-2 = -2

a = -2

From equation (1), we have;

-8 =  a - b + c = -2 - b - 1 = -3 - b

-8 + 3 = -b

-5 = -b

b = 5

The equation is therefore;

f(x) = -2·x² + 5·x - 1

The correct option is f(x) = -2·x² + 5·x - 1.

PLZ Help! I'm crying over this....

Answers

Answer:

D

Step-by-step explanation:

The correct option is the Car stops at a distance away from the starting point because the portion shows a constant function away from the starting point.

From the graph we can tell that the distance of the car is a function and it's a line

Answered by G a u t h m a t h

The perimeter of a triangle is 24 cm.If it’s area is 24cm2 and it’s one of the three sides is 10 cm,find the sides of remaining

Answers

Answer:

The other sides are 6cm and 8cm

Step-by-step explanation:

Let the other 2 sides be x and y.

Using herons formula, we have an equation

24=sqrt(12*2*(12-x)*(12-y))

Since the perimeter is given, we have x+y=24-10=14. Solving it we have x=6,8 and y=8,6.

Solve for y
4x + 7y +5

Answers

To find the slope, m, of the line represented algebraically by the given linear equation in two variables: 4x = 7y + 5, first put the given equation in the slope-intercept form of the equation of a straight line, i.e., y = mx + b, as follows:

4x = 7y + 5 (Given)

4x - 5 = 7y + 5 - 5

4x - 5 = 7y + 0

4x - 5 = 7y

7y = 4x - 5 since equality is symmetric, i.e., if a = b, then b = a.

(7y)/7 = (4x - 5)/7

(7/7)y = (4x)/7 - 5/7

(1)y = (4/7)x- 5/7

y = (4/7)x - 5/7

From the last step above, the given equation is now in the slope-intercept form of the equation of a straight line: y = mx + b, i.e., y = (4/7)x - 5/7; therefore, we can now see by observation, that the slope of the line is m = 4/7 which means that the line has a positive slope, i.e., it's > 0, which means that if the line were to be constructed on a Cartesian coordinate system, it would slope upward from left to right and would intersect the y-axis at the point (0, -5/7).

if one square yard of carpet costs $15.40, how much will 45.5 square yards cost? ( value only, no commas, dollar signs, symbols, or units)

Answers

Answer: 700.70 dollars

Work Shown:

1 square yard = 15.40 dollars

45.5*1 square yard = 45.5*15.40 dollars

45.5 square yards = 700.70 dollars

In the second step, I multiplied both sides by 45.5 to turn the "1 square yard" into "45.5" square yards.

A researcher performs a hypothesis test to test the claim that for a particular manufacturer, the mean weight of cereal in its 18 ounce boxes is less than 18 ounces. He uses the following hypotheses: H 0: μ = 18 vs H A: μ < 18 and finds a P-value of 0.01. Draw a conclusion about the cereal box weight at a significance level of 0.05.

Answers

Answer:

We conclude that the mean weight of cereal in its 18-ounce boxes is less than 18 ounces.

Step-by-step explanation:

We are given that a researcher performs a hypothesis test to test the claim that for a particular manufacturer, the mean weight of cereal in its 18-ounce boxes is less than 18 ounces.

Let [tex]\mu[/tex] = mean weight of cereal in its 18-ounce boxes.

So, Null Hypothesis, [tex]H_0[/tex] : [tex]\mu[/tex] = 18       {mean that the mean weight of cereal in its 18-ounce boxes is equal to 18 ounces}

Alternate Hypothesis, [tex]H_A[/tex] : [tex]\mu[/tex] < 18      {mean that the mean weight of cereal in its 18-ounce boxes is less than 18 ounces}

Also, it is given that the P-value is 0.01 and the level of significance is 0.05.

The decision rule based on the P-value is given by;

If the P-value of our test statistics is less than the level of significance, then we have sufficient evidence to reject our null hypothesis as our test statistics will fall in the rejection region.If the P-value of our test statistics is more than the level of significance, then we have insufficient evidence to reject our null hypothesis as our test statistics will not fall in the rejection region.

Here, clearly our P-value is less than the level of significance as 0.01 < 0.05, so we have sufficient evidence to reject our null hypothesis as our test statistics will fall in the rejection region.

Therefore, we conclude that the mean weight of cereal in its 18-ounce boxes is less than 18 ounces.

I need help It’s honors 7 math pls help

Answers

Answer:

411,600.000× 10^3 OR 4.11600000×10^8

Step-by-step explanation:

The expontent seems to be 3 so you move the decimal point over <this way 3

if it is 8 you move it over this way < 8

Answer:

411,600.000× 10^3 OR 4.11600000×10^8

Step-by-step explanation:

The expontent seems to be 3 so you move the decimal point over <this way 3

if it is 8 you move it over this way < 8

Is the given equation a quadratic equation? Explain. x(x−6)=−5

The equation is not a quadratic equation because there is no x2-term.

The equation is a quadratic equation because there is an x2-term.

The equation is not a quadratic equation because the expression is not equal to zero.

The equation is not a quadratic equation because there is a term with degree higher than 2.

I think the answer is A but im not sure.

Answers

Answer:

The equation is a quadratic equation because there is an x2-term.

Step-by-step explanation:

x(x−6)=−5

Distribute

x^2 -6x = -5

The equation is a quadratic equation because there is an x^2-term.

Answer:

Your required answer is option A.

Step-by-step explanation:

Here,

The given equation is;

x(x-6)=-5

now,

while finding x.

either or,

x=-5 (x-6)=-5

x= 1 (shifting-6 in next side)

now, the value of x is -5,1.

so, it's a quadratic equation.

( in quadratic equation the variable always has two values after solution)

Hope it helps..

What is the solution to the equation below? (round your answer two decimal places)
e^x=7.1

Answers

Answer:

Step-by-step explanation:

In order to undo that e, you need to take the natural log of both sides:

[tex]ln(e^x)=ln(7.1)[/tex] the ln and the e cancel each other out, leaving us with

x = ln(7.1) so

x = 1.96

Tori and Gavin were trying to solve the equation: (x+1)^2-3=13(x+1) 2 −3=13left parenthesis, x, plus, 1, right parenthesis, squared, minus, 3, equals, 13 Tori said, "I'll add 333 to both sides of the equation and solve using square roots." Gavin said, "I'll multiply (x+1)^2(x+1) 2 left parenthesis, x, plus, 1, right parenthesis, squared and rewrite the equation as x^2+2x+1-3=13x 2 +2x+1−3=13x, squared, plus, 2, x, plus, 1, minus, 3, equals, 13. Then I'll subtract 131313 from both sides, combine like terms, and solve using the quadratic formula with a=1a=1a, equals, 1, b=2b=2b, equals, 2, and c=-15c=−15c, equals, minus, 15."

Answers

The other answer is correct, its both !<3

Answer:

Both

Step-by-step explanation:

Both Tori and Gavin are correct, the two methods work. Completed this in Khan Academy, it's correct.

Other Questions
a) Provide equation of K of this reaction, use symbol " ^ " for exponents. That means 1000 = 10^3 and 1/100 is 10^(-2). b) How many moles of compound F will be produced if only 2 moles of compound C is available? describe or show your work. 3 A + 5 B +4 C 5 D +7 E + F A student with a body temperature of 37C holds a piece of ice that has atemperature of -8C.Which statement best explains what happens?1. Thermal energy transfers coldness from the student's hand to the ice.2. Heat transfers coldness from the ice to the student's hand.3. Heat transfers thermal energy from the student's hand to the ice.4. Thermal energy transfers heat from the student's hand to the ice. in this chapter, we meet Cinna, Katniss's stylist. Cinna is not like the other people in thecapitol. What makes him stand out? How does he help Katniss and Peeta to also standout among the other tributes? Why is this an important strategy? Bob and Carol want to hire Able Refinishing to sand and refinish the dining room floor to match the floor in the living room. Able charges $2.41 per square foot to sand and refinish a hardwood floor. The dining room is rectangular and measures 17 feet 10 inches by 11 feet 9 inches. Find the area of the dining room floor and the cost of the work. Methane (CH4) is the major component of natural gas. 40.0 grams of methane were placed in a commercial calorimeter and subjected to a combustion reaction. The reaction released 2800 kJ of energy.1. Compare this energy value to the energy values of paraffin and isopropanol. Is methane a good choice as a fuel? if pentagon OPQRS is dilated by a scale factor or ?from the origin to create O'P'Q'R'S: what is the ordered pair of point S'? Find the sum of the infinite geometric series -27, -9, -3, The ratio is /3 and u1 is -27 What is the product of 2p + q and -39 - 6p + 1?0 -12p2 - 6pq 4p - 32 + 1O-12p2- 12pq + 2p 392 +9- gp q + 12pq - 2p + 912p2 + 12pq +2p + 302 + 9 How many films does he have ? Please answer all 3 questions if you know it! The answers on the picture are not fully correct just random ones I clicked. Simply this rational expression.URGENT! PICTURE INCLUDED Which equation represents a parabola that has a focus of (0,0) and a directix of y = 2? Consider the functions F and G in the tables belowPLEASE HELP PLEASE poema de la naturaleza de 3 estrofas y cuatro versos que no sea de gogle The National Defense Education Act, which for the first time offered direct federal funding for higher education, was passed into law by Congress in 1957 in response to Group of answer choices inner-city riots, and the rise of feminist activist endeavors across the Midwest. the French defeat by Vietnamese forces at Dien Bien Phu. Soviet Premier Nikita Khrushchev's visit to the United States. the Soviet launch of the first artificial Earth satellite, Sputnik. SOMEBODY PLEASE HELP ME ON THIS ; DUE TODAY, ill mark u the brainliest When prices for homes rise, why might construction companies decide to build more homes?A. to increase employmentB. to make a profitC. to create a need for more resourcesD. to boost name recognition which is more important when building or buying a new home its location or its size essay ? Find the squares of the following:1)2a+1 2) 3b+2c 3) 2p+3q A fish is 80 cm below the surface of a pond. What is the apparent depth (in cm) when viewed from a position almost directly above the fish